注意这次交三次的作业,缺交的同学及时补交(邮箱:zzw@smail.nju.edu.cn)

9-24

4.2

  • (a) $\frac{1}{2}n(n-1)$, decreasing sequence

  • (b) increasing, $n-1$

    4.3

  • (a) 用循环不变式证明 ($E[j]$ is the maximum in $[0,j]$)

  • (b) 略

    4.4

  • (a) 可利用 4.3(a) 的循环不变式证明

  • 不影响

    4.23

注意需使用 Section 2.3.2 的数据结构

4.27

  • (a) 可以使用两个数组 $A,B$,基于归并时处于归并树结点深度的奇偶性,奇数时保存到 $A$,偶数时保存到 $B$(滚动数组)

  • (b) $n\log n$

    4.28

找中点:设置两个指针,一个步长为 1, 一个步长为 2,当快指针到达尾结点时,慢指针指向中间结点。该时间复杂度为 $O(n)$。由于 merge 也是 $O(n)$,时间复杂度不变为 $O(n\log n)$。空间复杂度 $O(n\log n)$,有 garbage collection 为 $O(n)$

9-28

1.27

$n\leq 25$ 算法一快

1.28

1.31

用符号之前应说明一下,$<$ 和 $=$ 都是在渐进复杂度关系下比较。最后一项可查阅斯特林公式

$\lg\lg n<\lg n=\ln n<(\lg n)^2<\sqrt{n}<n<n\lg n<n^{1+\epsilon}<n^2=n^2+\lg n<n^3<n-n^3+7n^5<2^{n-1}=2^n<e^n=n!$

1.32

反例:$2^n,n!$

1.33

根据定义($O$ 是否使用极限定义)不同,可以证明或举反例

  • 证明思路:$\forall f\in o(f),f\in O(f)-\Theta(f)$ and $\forall f\in O(f)-\Theta(f),f\in o(f)$
  • 举反例:找一个极限不存在的函数

3.7

  • (a) $O(n\log n)$

  • (b) $M(n)=n\log_2n-n+1$

    3.8

$O(n)$

3.10

Tips: 部分使用 Master Theorem 更加简单

  • (a) $O(\lg^2 n)$
  • (b) $O(n)$
  • (c) $O(n\lg n)$
  • (d) $O(n(\lg n)^2)$
  • (e) $O(n^2)$

大整数乘法

$T(n)=3T(\frac{m}{2})$,$T(n)=O(n^{\log_23})$

9-30

6.3

6.7

破坏黑高

6.8

6.9

15 个节点高位 4 的满二叉树

例:1-15 顺序插入,得到 8 4 12 14 10 6 2 15 13 11 9 7 5 3 1